Difference between revisions of "2021 AMC 10B Problems/Problem 1"

(Redirected page to 2021 AMC 12B Problems/Problem 1)
(Tag: New redirect)
(Removed redirect to 2021 AMC 12B Problems/Problem 1)
(Tag: Removed redirect)
Line 1: Line 1:
#redirect [[2021 AMC 12B Problems/Problem 1]]
+
==Problem==
 +
How many integer values of <math>x</math> satisfy <math>|x|<3\pi</math>?
 +
 
 +
<math>\textbf{(A)} ~9 \qquad\textbf{(B)} ~10 \qquad\textbf{(C)} ~18 \qquad\textbf{(D)} ~19 \qquad\textbf{(E)} ~20</math>
 +
 
 +
==Solution==
 +
Since <math>3\pi</math> is about <math>9.42</math>, we multiply 9 by 2 and add 1 to get <math> \boxed{\textbf{(D)}\ ~19} </math>~smarty101

Revision as of 18:31, 11 February 2021

Problem

How many integer values of $x$ satisfy $|x|<3\pi$?

$\textbf{(A)} ~9 \qquad\textbf{(B)} ~10 \qquad\textbf{(C)} ~18 \qquad\textbf{(D)} ~19 \qquad\textbf{(E)} ~20$

Solution

Since $3\pi$ is about $9.42$, we multiply 9 by 2 and add 1 to get $\boxed{\textbf{(D)}\ ~19}$~smarty101